7dcb65b3d004be2d2f73c7d9ea6a62ff8ea591ac
[course.git] / latex / problems / Serway_and_Jewett_8 / problem10.51.tex
1 \begin{problem*}{10.51}
2 An object with a mass $m=5.10\U{kg}$ is attached to the free end of a
3 light string wrapped around a reel of radius $R=0.250\U{m}$ and mass
4 $M=3.00\U{kg}$.  The reel is a solid disk, free to rotate in a
5 vertical plane about the horiizontal axis passing through its center
6 as shown in Figure~P10.51.  The suspended object is released from rest
7 $6.00\U{m}$ above the floor.  Determine \Part{a} the tension in the
8 string, \Part{b} the acceleration of the object, and \Part{c} the
9 speed with which the object hits the floor.  \Part{d} Verify your
10 answer to \Part{c} by using the isolated system (energy) model.
11 \end{problem*}
12
13 \begin{solution}
14 \Part{a}
15 Summing the forces and tourques,
16 \begin{align}
17   \sum F &= mg - T = ma \\
18   \sum \tau &= RT = I\alpha
19     = \p({\frac{1}{2}MR^2})\cdot\frac{a}{R}
20     = \frac{MRa}{2} \;.
21 \end{align}
22 Solving for tension
23 \begin{align}
24   a &= g - \frac{T}{m} \\
25   RT &= \frac{MRa}{2} \\
26   2T &= Ma \\
27   2T &= Mg - \frac{M}{m}T \\
28   T \p({2 + \frac{M}{m}}) &= Mg \\
29   T &= \frac{Mg}{2 + \frac{M}{m}} = \ans{11.4\U{N}} \;.
30 \end{align}
31
32 \Part{b}
33 Plugging back in for $a$,
34 \begin{equation}
35   a = g - \frac{T}{m} = 9.80\U{m/s$^2$} - \frac{11.4\U{N}}{5.10\U{kg}}
36     = \ans{7.57\U{m/s$^2$}}
37 \end{equation}
38
39 \Part{c}
40 Finding the floor-hitting speed is a constant acceleration problem
41 \begin{align}
42   v^2 &= v_0^2 + 2a\Delta x \\
43   v &= \sqrt{2a\Delta x} = \ans{9.53\U{m/s}}
44 \end{align}
45
46 \Part{d}
47 Conserving energy, the initial gravitational energy is converted into
48 linear and angular kinetic energies.
49 \begin{align}
50   E_i = mgh &= E_f = \frac{1}{2}mv^2 + \frac{1}{2}I\omega^2
51     = \frac{1}{2}v^2 + \frac{1}{2} \p({\frac{1}{2}MR^2}) \p({\frac{v}{R}})^2
52     = \frac{m}{2}mv^2 + \frac{M}{4} v^2
53     = \frac{2m + M}{4}v^2 \\
54   v^2 &= \frac{4mgh}{2m+M} \\
55   v &= \sqrt{\frac{4mgh}{2m+M}} = \ans{9.53\U{m/s$^2$}} \;,
56 \end{align}
57 which is the same speed we got in \Part{c}.
58 \end{solution}